Last visit was: 18 Nov 2025, 18:32 It is currently 18 Nov 2025, 18:32
Close
GMAT Club Daily Prep
Thank you for using the timer - this advanced tool can estimate your performance and suggest more practice questions. We have subscribed you to Daily Prep Questions via email.

Customized
for You

we will pick new questions that match your level based on your Timer History

Track
Your Progress

every week, we’ll send you an estimated GMAT score based on your performance

Practice
Pays

we will pick new questions that match your level based on your Timer History
Not interested in getting valuable practice questions and articles delivered to your email? No problem, unsubscribe here.
Close
Request Expert Reply
Confirm Cancel
User avatar
Sajjad1994
User avatar
GRE Forum Moderator
Joined: 02 Nov 2016
Last visit: 18 Nov 2025
Posts: 17,289
Own Kudos:
49,291
 [8]
Given Kudos: 6,179
GPA: 3.62
Products:
Posts: 17,289
Kudos: 49,291
 [8]
Kudos
Add Kudos
8
Bookmarks
Bookmark this Post
User avatar
Its_me_aka_ak
Joined: 16 Jul 2023
Last visit: 10 Jun 2025
Posts: 126
Own Kudos:
21
 [1]
Given Kudos: 310
Location: India
GPA: 3.46
Posts: 126
Kudos: 21
 [1]
1
Kudos
Add Kudos
Bookmarks
Bookmark this Post
User avatar
Sajjad1994
User avatar
GRE Forum Moderator
Joined: 02 Nov 2016
Last visit: 18 Nov 2025
Posts: 17,289
Own Kudos:
49,291
 [1]
Given Kudos: 6,179
GPA: 3.62
Products:
Posts: 17,289
Kudos: 49,291
 [1]
1
Kudos
Add Kudos
Bookmarks
Bookmark this Post
User avatar
Shruti56
Joined: 09 Jun 2022
Last visit: 29 Aug 2025
Posts: 2
Given Kudos: 5
Posts: 2
Kudos: 0
Kudos
Add Kudos
Bookmarks
Bookmark this Post
Please explain question 2

Posted from my mobile device
User avatar
Sajjad1994
User avatar
GRE Forum Moderator
Joined: 02 Nov 2016
Last visit: 18 Nov 2025
Posts: 17,289
Own Kudos:
Given Kudos: 6,179
GPA: 3.62
Products:
Posts: 17,289
Kudos: 49,291
Kudos
Add Kudos
Bookmarks
Bookmark this Post
Shruti56
Please explain question 2

Explained here: https://gmatclub.com/forum/in-her-1977- ... l#p3265769
User avatar
Its_me_aka_ak
Joined: 16 Jul 2023
Last visit: 10 Jun 2025
Posts: 126
Own Kudos:
21
 [1]
Given Kudos: 310
Location: India
GPA: 3.46
Posts: 126
Kudos: 21
 [1]
Kudos
Add Kudos
1
Bookmarks
Bookmark this Post
Sajjad1994
Its_me_aka_ak
plz explain q2
i believe that statement 2 is inferred by research of individual subjects and not the theory referred in question

Official Explanation

2. It can be inferred from the passage that the author believes which of the following about social justification theory?

Explanation

Analyze the question stem

This is a Weaken question. The correct answer will be the choice that makes the referenced position less likely to be correct.

Research

The highlighted portion describes how women placed in an environment in which females are perceived to be poor math students experience a drop in math performance. This is an example used to support the idea that societal expectations determine individual behavior.

Make a prediction

The implication is that the women are doing more poorly in math because they believe that is what their society expects of them. Any choice that suggests some other explanation for their drop in performance would weaken the position.

Evaluate the answer choices

If, as stated in (C), the female math students' declining performance was matched by similar declines among male students placed in the same environment, this would weaken the position that the women were simply trying to conform to the society's stereotype. No such stereotype exists for the males, so something else must be going on to explain the drop in performance. (C) is therefore the correct answer.

(A) focuses on study habits, not performance, and so has nothing to do with the claim.

(B) actually strengthens the claim by removing another possible explanation, namely that female performance drops because teachers are more strict with female students.

(D) is incorrect because improvement by men doesn't imply anything about why women do worse.

If anything, (E) strengthens the argument because it suggests that societal expectations are a factor that determines women's performance.

TAKEAWAY: Your prediction to a Weaken question will necessarily be general, as there's no way to know exactly what the test maker will present in the correct answer. However, you can have a good idea of what the correct answer must do in a general sense, such as present an alternative explanation.

Answer: C
sir actually you explained Q 1, i was talking about Q2 and attacking statement 2 referring that it can actually be inferred but not by the theory in question rather later in the final paragraph
User avatar
saurabhin0921
Joined: 02 Sep 2024
Last visit: 10 Apr 2025
Posts: 1
Own Kudos:
1
 [1]
Given Kudos: 21
Posts: 1
Kudos: 1
 [1]
1
Kudos
Add Kudos
Bookmarks
Bookmark this Post
You have mistyped Question 2 instead of Question 1, please can you post an explanation for Q2.
Sajjad1994
Its_me_aka_ak
plz explain q2
i believe that statement 2 is inferred by research of individual subjects and not the theory referred in question

Official Explanation

2. It can be inferred from the passage that the author believes which of the following about social justification theory?

Explanation

Analyze the question stem

This is a Weaken question. The correct answer will be the choice that makes the referenced position less likely to be correct.

Research

The highlighted portion describes how women placed in an environment in which females are perceived to be poor math students experience a drop in math performance. This is an example used to support the idea that societal expectations determine individual behavior.

Make a prediction

The implication is that the women are doing more poorly in math because they believe that is what their society expects of them. Any choice that suggests some other explanation for their drop in performance would weaken the position.

Evaluate the answer choices

If, as stated in (C), the female math students' declining performance was matched by similar declines among male students placed in the same environment, this would weaken the position that the women were simply trying to conform to the society's stereotype. No such stereotype exists for the males, so something else must be going on to explain the drop in performance. (C) is therefore the correct answer.

(A) focuses on study habits, not performance, and so has nothing to do with the claim.

(B) actually strengthens the claim by removing another possible explanation, namely that female performance drops because teachers are more strict with female students.

(D) is incorrect because improvement by men doesn't imply anything about why women do worse.

If anything, (E) strengthens the argument because it suggests that societal expectations are a factor that determines women's performance.

TAKEAWAY: Your prediction to a Weaken question will necessarily be general, as there's no way to know exactly what the test maker will present in the correct answer. However, you can have a good idea of what the correct answer must do in a general sense, such as present an alternative explanation.

Answer: C
User avatar
rishabh1105
Joined: 25 Mar 2025
Last visit: 20 Jun 2025
Posts: 21
Own Kudos:
9
 [2]
Given Kudos: 2
Posts: 21
Kudos: 9
 [2]
2
Kudos
Add Kudos
Bookmarks
Bookmark this Post
H2. It can be inferred from the passage that the author believes which of the following about social justification theory?

I. It provides a complete picture of how individuals think of their social system.
II. It states that social reform can be repudiated by those who might most benefit from it.
III. It is used primarily to understand gender relations.

A. I Only
B. II Only
C. III Only
D. I and II Only
E. II and III Only

Lets talk about each of the statements-
I. It provides a complete picture of how individuals think of their social system. - The first line of 3rd paragraph mentions "What is not known is whether or not this intransigence in the face of facts would hold true for historically advantaged groups as well." which means there are missing elements in SJT.
So the statement cannot be inferred.

II. It states that social reform can be repudiated by those who might most benefit from it. -
"But in small-scale individual testing in which men were presented with evidence that they believed removed some of their advantaged status, the subjects actually ended up with an increase in self-esteem related to their reduced advantage, embracing the reduction and taking pride in it."
Inference can be drawn

III. It is used primarily to understand gender relations.
SJT talks about "System justification theory (SJT) posits that even those disadvantaged by a social system have a tendency to defend and support it, contributing to the stasis of the status quo." - gender examples in the passage are only one type of example shown there can be other groups in the social system that are advantaged/disadvantaged

Hence answer is B
User avatar
Sajjad1994
User avatar
GRE Forum Moderator
Joined: 02 Nov 2016
Last visit: 18 Nov 2025
Posts: 17,289
Own Kudos:
49,291
 [1]
Given Kudos: 6,179
GPA: 3.62
Products:
Posts: 17,289
Kudos: 49,291
 [1]
1
Kudos
Add Kudos
Bookmarks
Bookmark this Post
Official Explanation

2. It can be inferred from the passage that the author believes which of the following about social justification theory?

Analyze the question stem

This is an Inference question, as indicated by the word "inferred." The answer will be the statement or statements that have direct support from the passage.
Research

"Social justification theory" is not specific enough to direct any research at this point, as the theory is discussed throughout the second and third paragraphs. Just review the passage map notes for those two paragraphs and proceed to the choices. Be prepared to do further research about each choice.
Make a prediction

The answers to Inference questions—particularly those without strong research clues—are usually not predictable. Keep in mind from the passage map that the theory shows that the disadvantaged tend to defend and support the status quo, but that it isn't clear whether advantaged groups do the same. Correct answers will likely align with this idea. Then review each choice and ask whether there is direct support for that choice in the passage, researching the text as needed.

Evaluate the answer choices

(B) is correct, because it is essentially a paraphrase of the statement in the passage that "even those disadvantaged by a social system have a tendency to
defend and support it."


(A) is not supported by the passage and is therefore incorrect. One should always be suspicious of extreme language in Inference question choices, such as (A)'s "complete." As the third paragraph discusses, there is less clarity about advantaged groups. The third paragraph even begins with language of doubt: "What is not known..." Thus, the picture provided by SJT is incomplete, and (A) must be rejected.

(C) Rejected.

Answer: B
Moderators:
GMAT Club Verbal Expert
7445 posts
GMAT Club Verbal Expert
234 posts
GRE Forum Moderator
17289 posts
188 posts